Mathcenter Forum  

Go Back   Mathcenter Forum > คณิตศาสตร์โอลิมปิก และอุดมศึกษา > อสมการ
สมัครสมาชิก คู่มือการใช้ รายชื่อสมาชิก ปฏิทิน ข้อความวันนี้

ตั้งหัวข้อใหม่ Reply
 
เครื่องมือของหัวข้อ ค้นหาในหัวข้อนี้
  #31  
Old 01 มีนาคม 2005, 20:29
gon's Avatar
gon gon ไม่อยู่ในระบบ
ผู้พิทักษ์กฎขั้นสูง
 
วันที่สมัครสมาชิก: 29 มีนาคม 2001
ข้อความ: 4,608
gon is on a distinguished road
Smile

เดี๋ยวแก้ไขก่อนเบลออย่างแรง.

01 มีนาคม 2005 23:10 : ข้อความนี้ถูกแก้ไขแล้ว 1 ครั้ง, ครั้งล่าสุดโดยคุณ gon
ตอบพร้อมอ้างอิงข้อความนี้
  #32  
Old 02 มีนาคม 2005, 11:07
gools's Avatar
gools gools ไม่อยู่ในระบบ
บัณฑิตฟ้า
 
วันที่สมัครสมาชิก: 26 เมษายน 2004
ข้อความ: 390
gools is on a distinguished road
Post

ขออภัยครับ พอดีรีบไปหน่อย
แก้ไขแล้วนะครับ
ตอบพร้อมอ้างอิงข้อความนี้
  #33  
Old 02 มีนาคม 2005, 20:57
gools's Avatar
gools gools ไม่อยู่ในระบบ
บัณฑิตฟ้า
 
วันที่สมัครสมาชิก: 26 เมษายน 2004
ข้อความ: 390
gools is on a distinguished road
Post

คือผมเจอคำถามคล้ายกับข้อ 11. ในartofproblemsolving อยู่ในรูป \(\sum \frac{x^{4}}{x^{3}+y^{3}}\) แล้วก็มีคนที่ชื่อ rodja เสนอให้พิสูจน์ในกรณีทั่วไป โดยเปลี่ยนคำถามใหม่เป็น
ให้ \(x,y,z>0\) และ \(a \geq b > 0\) จงพิสูจน์ว่า \[\frac{x^a}{y^b+z^b}+\frac{y^a}{z^b+x^b}+\frac{z^a}{x^b+y^b} \geq \frac{x^{a-b}+y^{a-b}+z^{a-b}}{2}\] แล้วก็มีคนตอบเข้ามาว่า
It is not symmetric at all.Thus, what rodja posted is NOT a generalization.In fact, rodja's problem is much simpler.
หมายความว่ายังไงครับ(ไม่ค่อยเชี่ยวอังกฤษ )

02 มีนาคม 2005 21:38 : ข้อความนี้ถูกแก้ไขแล้ว 2 ครั้ง, ครั้งล่าสุดโดยคุณ gools
ตอบพร้อมอ้างอิงข้อความนี้
  #34  
Old 04 มีนาคม 2005, 10:36
gon's Avatar
gon gon ไม่อยู่ในระบบ
ผู้พิทักษ์กฎขั้นสูง
 
วันที่สมัครสมาชิก: 29 มีนาคม 2001
ข้อความ: 4,608
gon is on a distinguished road
Smile

ช่วงนี้ยังไม่มี้เวลามาคิดต่อปัญหาที่เหลืออยู่บ้างเลยครับ. วุ่นจริง ๆ เดี๋ยวอีก 3 - 4 วัน น่าจะคงว่างบ้างแล้วจะมาร่วมแก้ต่อ.
ตอบพร้อมอ้างอิงข้อความนี้
  #35  
Old 05 มีนาคม 2005, 09:24
nooonuii nooonuii ไม่อยู่ในระบบ
ผู้พิทักษ์กฎทั่วไป
 
วันที่สมัครสมาชิก: 25 พฤษภาคม 2001
ข้อความ: 6,408
nooonuii is on a distinguished road
Post

โจทย์ข้อ 15 ของน้อง gool นี่ยากมากทีเดียวครับ เป็นโจทย์ IMO'2001 ผมก็คิดไม่ออกเหมือนกัน แต่มีเฉลยที่สสวท.ทำไว้ครับ เป็นวิธีพิสูจน์ที่สวยงามหมดจดจริงๆครับ

จะพิสูจน์ว่า
\[ \large{ \frac{a}{\sqrt{a^2+8bc}} \geq \frac{a^{4/3}}{a^{4/3} + b^{4/3} + c^{4/3}} } \]

ซึ่งสมมูลกับ
\[ (b^{4/3} + c^{4/3})(a^{4/3} + a^{4/3} + b^{4/3} + c^{4/3}) \geq 8a^{2/3}bc \]

ซึ่งพิสูจน์โดยใช้ AM-GM กับทั้งสองเทอมก็จะได้อสมการที่ต้องการครับ

เข้าไปดูในเวบอื่นรู้สึกว่าจะมีวิธีที่ใช้ Cauchy's inequality ด้วยครับ แต่ผมว่ายากกว่าวิธีนี้

สำหรับข้ออื่นยังมีคนแอบคิดอยู่รึปล่าวครับเนี่ย
__________________
site:mathcenter.net คำค้น

05 มีนาคม 2005 09:40 : ข้อความนี้ถูกแก้ไขแล้ว 3 ครั้ง, ครั้งล่าสุดโดยคุณ nooonuii
ตอบพร้อมอ้างอิงข้อความนี้
  #36  
Old 05 มีนาคม 2005, 19:49
aaaa's Avatar
aaaa aaaa ไม่อยู่ในระบบ
ลมปราณบริสุทธิ์
 
วันที่สมัครสมาชิก: 01 มกราคม 2005
ข้อความ: 109
aaaa is on a distinguished road
Talking

ข้อ 15 ยากมากทีเดียวครับ แต่มีวิธีคิดแบบอื่นเหมือนกัน ผมยังบอกไม่ได้ตอนนี้ เพราะเร็วๆนี้จะมีข้อสอบแข่งขัน (เป็นโจทย์ inequality) ที่ใช้แนวคิดใหม่นี้
ตอบพร้อมอ้างอิงข้อความนี้
  #37  
Old 07 มีนาคม 2005, 15:03
gon's Avatar
gon gon ไม่อยู่ในระบบ
ผู้พิทักษ์กฎขั้นสูง
 
วันที่สมัครสมาชิก: 29 มีนาคม 2001
ข้อความ: 4,608
gon is on a distinguished road
Smile

ข้อ 14 นี่มีอะไรแปลก ๆ หรือเปล่าครับ.

โดยอสมการ Jensen : สำหรับทุกจำนวนจริงบวก \(a_1, a_2, \cdots , a_n , p, q\) และ \(p < q, n \geq 2 \Rightarrow \sum_{i=1}^{n}(a_i^p)^{\frac{1}{p}} > \sum_{i=1}^{n}(a_i^q)^{\frac{1}{q}} \)

\(\Rightarrow (a^5 + b^5 + c^5)^{\frac{1}{5}} > (a^7 + b^7 + c^7)^{\frac{1}{7}} \Rightarrow (a^5 + b^5 + c^5)^7 > (a^7 + b^7 + c^7)^5 \)
\(\Rightarrow 7\ln(a^5 + b^5 + c^5) > 5\ln(a^7 + b^7 + c^7)\Rightarrow \quad \displaystyle{ \frac{\ln(a^5 + b^5 + c^5)}{\ln(a^7 + b^7 + c^7)} > \frac{5}{7} }\)
ตอบพร้อมอ้างอิงข้อความนี้
  #38  
Old 07 มีนาคม 2005, 15:24
warut warut ไม่อยู่ในระบบ
กระบี่ไร้สภาพ
 
วันที่สมัครสมาชิก: 24 พฤศจิกายน 2001
ข้อความ: 1,627
warut is on a distinguished road
Post

ผมเดาว่า ln(a7 + b7 + c7) มันน้อยกว่า 0 น่ะครับ เวลาย้ายข้างเครื่องหมายเลยต้องกลับด้านด้วย
ตอบพร้อมอ้างอิงข้อความนี้
  #39  
Old 09 มีนาคม 2005, 02:58
nooonuii nooonuii ไม่อยู่ในระบบ
ผู้พิทักษ์กฎทั่วไป
 
วันที่สมัครสมาชิก: 25 พฤษภาคม 2001
ข้อความ: 6,408
nooonuii is on a distinguished road
Post

อ่า...ใช่แล้วครับ พอใส่ ln แล้วมันจะติดลบเลยต้องกลับเครื่องหมาย ส่วนอีกข้างใบ้ให้ว่าใช้ Holder's Inequality ครับ สำหรับข้ออื่นๆ ยังมีใครคิดอยู่รึปล่าวครับ
__________________
site:mathcenter.net คำค้น
ตอบพร้อมอ้างอิงข้อความนี้
  #40  
Old 11 มีนาคม 2005, 18:16
gools's Avatar
gools gools ไม่อยู่ในระบบ
บัณฑิตฟ้า
 
วันที่สมัครสมาชิก: 26 เมษายน 2004
ข้อความ: 390
gools is on a distinguished road
Post

ข้อ 8 จะพิสูจน์ในกรณีทั่วไปนะครับ
ให้จำนวนนับ \(n \geq 3\) และ \(a_1,...,a_n\) เป็นจำนวนที่ไม่ใช่จำนวนจริงลบ ให้
\[(n-2) \sum_{i<j} a_i a_j + \sum_{i<j<k} a_i a_j a_k = \frac{2n(n-1)(n-2)}{3}\]
จงพิสูจน์ว่า
\[\sum_{1 \leq i \leq n} a_i \geq \frac{2}{n-1} \sum_{i<j} a_i a_j\]
จะพิสูจน์ในกรณี \(n=3\)
ให้ \(ab+bc+ca+abc=4\)
ต้องพิสูจน์ว่า \(a+b+c \geq ab+bc+ca\)
\[
\begin{array}{rcl}
ab+bc+ca+abc&=&4\qquad \\
\text{ให้ }a \leq b \leq c \text{ โดยไม่เสียนัยทั่วไป จะได้ว่า }3a^{2}+a^{3} &\leq& 4 \\
\therefore\qquad a &\leq& 1 \\
\text{ทำให้มี }c \geq 1 \text{ และ }b&=&\frac{4-ac}{a+b+ac} \\
\text{ แทนค่า }b\text{ ใน }a+b+c-ab-bc-ca\text{ จะได้ว่า} \\
&&a+b+c-ab-bc-ca \\
&=&a(a+c+ac)+4-ac+c{a+c+ac}-a(4-ac)-c(4-ac)-ac(a+c+ac) \\
&=&(a+c-2)^{2}+ac(c-1)(1-a) \geq 0 \qquad \therefore\text{ เป็นจริง} \\
\end{array}
\]
จะพิสูจน์ในกรณี \(n=4\)
ให้ \(f(x)=x^{4}-Ax^{3}+Bx^{2}-Cx+D=0\)
ให้ \(a,b,c,d\) เป็นคำตอบของสมการ จะได้ว่า
\[
A=a+b+c+d,B=ab+ac+ad+bc+bd+cd,C=abc+bcd+cda+dab
\]
diff ทั้ง 2 ข้างของสมการ จะได้ว่า \(4x^{3}-3Ax^{2}+2Bx-C=0\)
ให้ \(\alpha,\beta,\gamma\) เป็นคำตอบของสมการ
จะได้ว่า \(\alpha+\beta+\gamma=\frac{3A}{4},\alpha\beta+\beta\gamma+\gamma\alpha=\frac{B}{2},\alpha\beta\gamma=\frac{C}{4}\)
\(\alpha\beta+\beta\gamma+\gamma\alpha+\alpha\beta\gamma=\frac{2B+C}{4}=4\qquad\qquad\text{ เนื่องจาก }(2B+C=16)\)
เนื่องจาก \(\alpha\beta+\beta\gamma+\gamma\alpha+\alpha\beta\gamma=4\)
ดังนั้น \(\alpha+\beta+\gamma=\frac{3A}{4} \geq \alpha\beta+\beta\gamma+\gamma\alpha=\frac{B}{2}\)
จะได้ว่า \(A \geq \frac{2}{3}B\)
เนื่องจาก \(A=a+b+c+d \text{ และ } B=ab+ac+ad+bc+bd+cd\)
ดังนั้นอสมการเป็นจริงใน \(n=4\)

เมื่อพิสูจน์กรณี \(n=5 \text{ ก็สร้าง } g(x)\) ขึ้นมาแล้วพิสูจน์โดยใช้วิธีเดียวกับกรณี \(n=4\) ครับ ทำอย่างนี้ไปเรื่อยๆ โดยใช้วิธีเดิมก็จะได้ว่าอสมการข้างต้นที่เราต้องการพิสูจน์เป็นจริง

11 มีนาคม 2005 22:03 : ข้อความนี้ถูกแก้ไขแล้ว 3 ครั้ง, ครั้งล่าสุดโดยคุณ gools
ตอบพร้อมอ้างอิงข้อความนี้
  #41  
Old 11 มีนาคม 2005, 18:29
gools's Avatar
gools gools ไม่อยู่ในระบบ
บัณฑิตฟ้า
 
วันที่สมัครสมาชิก: 26 เมษายน 2004
ข้อความ: 390
gools is on a distinguished road
Post

อ่า รู้สึกว่าวิธีนี้จะใช้ได้กับ n=4 เท่านั้นนะครับ
ตอบพร้อมอ้างอิงข้อความนี้
  #42  
Old 11 มีนาคม 2005, 18:52
gools's Avatar
gools gools ไม่อยู่ในระบบ
บัณฑิตฟ้า
 
วันที่สมัครสมาชิก: 26 เมษายน 2004
ข้อความ: 390
gools is on a distinguished road
Post

ได้แล้วครับ
กรณีทั่วไป
ให้ \(f(x)=x^{n}-Ax^{n-1}+Bx^{n-2}-Cx^{n-3}+Dx^{n-4}-...=0\)
diff ไปเรื่อยๆ จะได้ว่า
\[\begin{array}{rcl}\frac{n!}{4!}x^{4}-\frac{(n-1)!}{3!}Ax^{3}+\frac{(n-2)!}{2!}Bx^{2}-(n-3)!Cx+(n-4)!D&=&0 \\
x^{4}-\frac{4}{n}Ax^{3}+\frac{12}{n(n-1)}Bx^{2}-\frac{24}{n(n-1)(n-2)}Cx+\frac{24}{n(n-1)(n-2)(n-3)}D&=&0 \qquad (1) \\
\text{และ }\qquad\frac{n!}{3!}x^{3}-\frac{(n-1)!}{2!}Ax^{2}+\frac{(n-2)!}Bx-(n-3)!Cx&=&0 \\
x^{3}-\frac{3}{n}Ax^{2}+\frac{6}{n(n-1)}Bx-\frac{9}{n(n-1)(n-2)}C&=&0
\end{array}
\]
แทนสัมประสิทธิ์ใน \((1)\) ด้วย \(p,q \text{ และ } r\)
จะได้ว่า
\[\begin{array}{rcl}x^{3}-\frac{3}{n}Ax^{2}+\frac{q}{2}x-\frac{r}{4}&=&0 \\
\text{เนื่องจาก }\qquad \frac{q}{2}x+\frac{r}{4}&=&\frac{2q+r}{4}=\frac{16}{4}=4\qquad (2q+r=16) \\
\text{ดังนั้น }\qquad\qquad \frac{3}{n}A &\geq& \frac{6}{n(n-1)}B \\
A &\geq& \frac{2}{n-1}B
\end{array}
\]

11 มีนาคม 2005 22:32 : ข้อความนี้ถูกแก้ไขแล้ว 4 ครั้ง, ครั้งล่าสุดโดยคุณ gools
ตอบพร้อมอ้างอิงข้อความนี้
  #43  
Old 15 มีนาคม 2005, 15:43
gools's Avatar
gools gools ไม่อยู่ในระบบ
บัณฑิตฟ้า
 
วันที่สมัครสมาชิก: 26 เมษายน 2004
ข้อความ: 390
gools is on a distinguished road
Post

เพิ่งกลับจากค่ายมาครับ
ข้างบนมีอะไรผิดก็บอกนะครับ(อย่างเช่น diff เกิน 2 ทีไม่ได้เป็นต้น )
เหลือข้อ 12 ข้อเดียวครับ
ตอบพร้อมอ้างอิงข้อความนี้
ตั้งหัวข้อใหม่ Reply



กฎการส่งข้อความ
คุณ ไม่สามารถ ตั้งหัวข้อใหม่ได้
คุณ ไม่สามารถ ตอบหัวข้อได้
คุณ ไม่สามารถ แนบไฟล์และเอกสารได้
คุณ ไม่สามารถ แก้ไขข้อความของคุณเองได้

vB code is On
Smilies are On
[IMG] code is On
HTML code is Off
ทางลัดสู่ห้อง


เวลาที่แสดงทั้งหมด เป็นเวลาที่ประเทศไทย (GMT +7) ขณะนี้เป็นเวลา 04:45


Powered by vBulletin® Copyright ©2000 - 2024, Jelsoft Enterprises Ltd.
Modified by Jetsada Karnpracha